GRE网络课堂学习笔记-逻辑[7]

来源:GRE考试    发布时间:2013-01-03    GRE考试辅导视频    评论

 

第四讲 规则题

1. Questions 12-17

A hobbyist is stocking her aquarium with exactly three fish of different types and with exactly two species of paints. The only fish under consideration are a G , an H , a J , a K and an L and the only kinds of plants under consideration are of the species W , X , Y , and Z . She will observe the following conditions:
If she selects the G, she can select neither the H nor a Y.
She cannot select the H unless she selects the K.
She cannot select the J unless she selects a W.
If she selects the K, she must select an X.

(12). Which one of the following is an acceptable selection of fish and plants for the aquarium?

   Fish    Plants
(A) G, H, K   W, Y
(B) G, J, K   W, X
(C) G, J, L   X, Z
(D) H, J, L   W, Z
(E) H, K, L   Y, Z
(13). If the hobbyist selects the H, which one of the following must also be true?

(A) She selects at least one W.
(B) She selects at least one X.
(C) She selects the J, but no Y's.
(D) She selects the K ,but no X's.
(E) She selects at least one X, but no Y's.

(14). If the hobbyist selects both X's and Z's, which one of the following could be the group of fish she selects?

(A) G, H, K
(B) G, J, K
(C) G, K, L
(D) H, J, L
(E) J, K, L

(15). The hobbyist could select any of the following groups of fish for the aquarium EXCEPT:

(A) G, K, L
(B) H, J, K
(C) H, J, L
(D) H, K, L
(E) J, K, L

(16). If the hobbyist selects a Y, which one of the following must be the group of fish she selects?

(A) G, H, K
(B) H, J, K
(C) H, J, L
(D) H, K, L
(E) J, K, L

(17). The hobbyist could select ant of the following plant combinations EXCEPT:

(A) W and X
(B) W and Y
(C) W and Z
(D) X and Y
(E) X and Z

KEYS: BBCCDB

视频学习

我考网版权与免责声明

① 凡本网注明稿件来源为"原创"的所有文字、图片和音视频稿件,版权均属本网所有。任何媒体、网站或个人转载、链接转贴或以其他方式复制发表时必须注明"稿件来源:我考网",违者本网将依法追究责任;

② 本网部分稿件来源于网络,任何单位或个人认为我考网发布的内容可能涉嫌侵犯其合法权益,应该及时向我考网书面反馈,并提供身份证明、权属证明及详细侵权情况证明,我考网在收到上述法律文件后,将会尽快移除被控侵权内容。

最近更新

社区交流

考试问答